Can the same recursion pattern define several sequences of primes?












6












$begingroup$


First, some definitions:



Let $A$ be a shorthand for an infinite sequence
$( a(1), a(2), a(3), dots )$
of positive integers, likewise $B$ is an infinite sequence of positive integers
$( b(1), b(2), b(3) , dots)$,
and C is an increasing infinite sequence of positive integers
$( c(0), c(1), c(2), c(3), dots)$. Call $c(0)$ the initial value of $C$.



We say that the pair $(A,B)$ is a recursion pattern for $C$ if for each
positive integer n we have
$c(n) = a(n) * c(n-1) + b(n)$. Given a recursion pattern $(A,B)$, and an
initial value for $C$, we can reconstruct all the sequence $C$.



Clearly, for any increasing infinite sequence $C$ we can find a pair $(A,B)$
which is a recursion pattern for $C$, simply by taking all the $a(n)$ to
be equal to $1$, and defining the $b(n)$ as required. If the sequence $C$ is
increasing quickly, there will be many pairs $(A,B)$ which will be
recursion patterns for $C$.



In particular, for any increasing sequence of primes, we can find
recursion patterns, many of them if the sequence grows quickly.



Now the question: can a pair $(A,B)$ be a recursion pattern for more than
one sequence of primes? In other words, given $A$ and $B$ as a recursion
pattern, is it possible there is more than one initial value for $c(0)$
which will lead to an infinite sequence of primes?



Comments:
If the answer is yes, and we can prove that, I would expect the proof
to be quite difficult. Maybe a conditional proof would be possible.
On the other hand, it may be elementary to show that the answer is no.
This question was written up by Moshe Newman.










share|cite|improve this question











$endgroup$








  • 1




    $begingroup$
    I think a "no" answer would be more difficult than you expect; I think this could be seen as one weakening of the twin prime conjecture - if the twin prime conjecture is true, then set $A$ to be $(1, 1, dots)$, set $B$ to be the sequential differences of the larger of the twin primes, set $c(0) = 3, c'(0) = 5$. So a "no" would in fact also be a disproof of the twin prime conjecture.
    $endgroup$
    – user44191
    yesterday
















6












$begingroup$


First, some definitions:



Let $A$ be a shorthand for an infinite sequence
$( a(1), a(2), a(3), dots )$
of positive integers, likewise $B$ is an infinite sequence of positive integers
$( b(1), b(2), b(3) , dots)$,
and C is an increasing infinite sequence of positive integers
$( c(0), c(1), c(2), c(3), dots)$. Call $c(0)$ the initial value of $C$.



We say that the pair $(A,B)$ is a recursion pattern for $C$ if for each
positive integer n we have
$c(n) = a(n) * c(n-1) + b(n)$. Given a recursion pattern $(A,B)$, and an
initial value for $C$, we can reconstruct all the sequence $C$.



Clearly, for any increasing infinite sequence $C$ we can find a pair $(A,B)$
which is a recursion pattern for $C$, simply by taking all the $a(n)$ to
be equal to $1$, and defining the $b(n)$ as required. If the sequence $C$ is
increasing quickly, there will be many pairs $(A,B)$ which will be
recursion patterns for $C$.



In particular, for any increasing sequence of primes, we can find
recursion patterns, many of them if the sequence grows quickly.



Now the question: can a pair $(A,B)$ be a recursion pattern for more than
one sequence of primes? In other words, given $A$ and $B$ as a recursion
pattern, is it possible there is more than one initial value for $c(0)$
which will lead to an infinite sequence of primes?



Comments:
If the answer is yes, and we can prove that, I would expect the proof
to be quite difficult. Maybe a conditional proof would be possible.
On the other hand, it may be elementary to show that the answer is no.
This question was written up by Moshe Newman.










share|cite|improve this question











$endgroup$








  • 1




    $begingroup$
    I think a "no" answer would be more difficult than you expect; I think this could be seen as one weakening of the twin prime conjecture - if the twin prime conjecture is true, then set $A$ to be $(1, 1, dots)$, set $B$ to be the sequential differences of the larger of the twin primes, set $c(0) = 3, c'(0) = 5$. So a "no" would in fact also be a disproof of the twin prime conjecture.
    $endgroup$
    – user44191
    yesterday














6












6








6





$begingroup$


First, some definitions:



Let $A$ be a shorthand for an infinite sequence
$( a(1), a(2), a(3), dots )$
of positive integers, likewise $B$ is an infinite sequence of positive integers
$( b(1), b(2), b(3) , dots)$,
and C is an increasing infinite sequence of positive integers
$( c(0), c(1), c(2), c(3), dots)$. Call $c(0)$ the initial value of $C$.



We say that the pair $(A,B)$ is a recursion pattern for $C$ if for each
positive integer n we have
$c(n) = a(n) * c(n-1) + b(n)$. Given a recursion pattern $(A,B)$, and an
initial value for $C$, we can reconstruct all the sequence $C$.



Clearly, for any increasing infinite sequence $C$ we can find a pair $(A,B)$
which is a recursion pattern for $C$, simply by taking all the $a(n)$ to
be equal to $1$, and defining the $b(n)$ as required. If the sequence $C$ is
increasing quickly, there will be many pairs $(A,B)$ which will be
recursion patterns for $C$.



In particular, for any increasing sequence of primes, we can find
recursion patterns, many of them if the sequence grows quickly.



Now the question: can a pair $(A,B)$ be a recursion pattern for more than
one sequence of primes? In other words, given $A$ and $B$ as a recursion
pattern, is it possible there is more than one initial value for $c(0)$
which will lead to an infinite sequence of primes?



Comments:
If the answer is yes, and we can prove that, I would expect the proof
to be quite difficult. Maybe a conditional proof would be possible.
On the other hand, it may be elementary to show that the answer is no.
This question was written up by Moshe Newman.










share|cite|improve this question











$endgroup$




First, some definitions:



Let $A$ be a shorthand for an infinite sequence
$( a(1), a(2), a(3), dots )$
of positive integers, likewise $B$ is an infinite sequence of positive integers
$( b(1), b(2), b(3) , dots)$,
and C is an increasing infinite sequence of positive integers
$( c(0), c(1), c(2), c(3), dots)$. Call $c(0)$ the initial value of $C$.



We say that the pair $(A,B)$ is a recursion pattern for $C$ if for each
positive integer n we have
$c(n) = a(n) * c(n-1) + b(n)$. Given a recursion pattern $(A,B)$, and an
initial value for $C$, we can reconstruct all the sequence $C$.



Clearly, for any increasing infinite sequence $C$ we can find a pair $(A,B)$
which is a recursion pattern for $C$, simply by taking all the $a(n)$ to
be equal to $1$, and defining the $b(n)$ as required. If the sequence $C$ is
increasing quickly, there will be many pairs $(A,B)$ which will be
recursion patterns for $C$.



In particular, for any increasing sequence of primes, we can find
recursion patterns, many of them if the sequence grows quickly.



Now the question: can a pair $(A,B)$ be a recursion pattern for more than
one sequence of primes? In other words, given $A$ and $B$ as a recursion
pattern, is it possible there is more than one initial value for $c(0)$
which will lead to an infinite sequence of primes?



Comments:
If the answer is yes, and we can prove that, I would expect the proof
to be quite difficult. Maybe a conditional proof would be possible.
On the other hand, it may be elementary to show that the answer is no.
This question was written up by Moshe Newman.







nt.number-theory






share|cite|improve this question















share|cite|improve this question













share|cite|improve this question




share|cite|improve this question








edited yesterday









user44191

2,9681428




2,9681428










asked yesterday









David S. NewmanDavid S. Newman

924520




924520








  • 1




    $begingroup$
    I think a "no" answer would be more difficult than you expect; I think this could be seen as one weakening of the twin prime conjecture - if the twin prime conjecture is true, then set $A$ to be $(1, 1, dots)$, set $B$ to be the sequential differences of the larger of the twin primes, set $c(0) = 3, c'(0) = 5$. So a "no" would in fact also be a disproof of the twin prime conjecture.
    $endgroup$
    – user44191
    yesterday














  • 1




    $begingroup$
    I think a "no" answer would be more difficult than you expect; I think this could be seen as one weakening of the twin prime conjecture - if the twin prime conjecture is true, then set $A$ to be $(1, 1, dots)$, set $B$ to be the sequential differences of the larger of the twin primes, set $c(0) = 3, c'(0) = 5$. So a "no" would in fact also be a disproof of the twin prime conjecture.
    $endgroup$
    – user44191
    yesterday








1




1




$begingroup$
I think a "no" answer would be more difficult than you expect; I think this could be seen as one weakening of the twin prime conjecture - if the twin prime conjecture is true, then set $A$ to be $(1, 1, dots)$, set $B$ to be the sequential differences of the larger of the twin primes, set $c(0) = 3, c'(0) = 5$. So a "no" would in fact also be a disproof of the twin prime conjecture.
$endgroup$
– user44191
yesterday




$begingroup$
I think a "no" answer would be more difficult than you expect; I think this could be seen as one weakening of the twin prime conjecture - if the twin prime conjecture is true, then set $A$ to be $(1, 1, dots)$, set $B$ to be the sequential differences of the larger of the twin primes, set $c(0) = 3, c'(0) = 5$. So a "no" would in fact also be a disproof of the twin prime conjecture.
$endgroup$
– user44191
yesterday










2 Answers
2






active

oldest

votes


















4












$begingroup$

The answer is yes, there are "recursion patterns" which result in sequences of primes for more than one choice of $c(0)$. In fact, if you are given any two "starting points", it is possible to find such sequences. The proof comes from the pigeonhole principle.



Start with $c(0), c'(0)$. These are two prime numbers with difference $d(1) := c'(0) - c(0)$. Then as the set of prime numbers is infinite, there is some pair $p_1 < p'_1$ such that $p_1 equiv p'_1 (text{mod }d)$. Let $a(1) = frac{p'_1 - p_1}{d(1)}$; let $b(1) = p_1 - a(1) c(0)$. (Note: I've slightly cheated here, and will explain the cheat at the end). Then $c(1) = a(1) c(0) + b(1) = a(1) c(0) + p_1 - a(1) c(0) = p_1$, while $c'(1) = a(1) c(0) + b(1) = frac{p'_1 - p_1}{c'(0) - c(0)} c'(0) + p_1 - frac{p'_1 - p_1}{c'(0) - c(0)} c(0) = p'_1$, as desired.



This is a pattern that can be repeated in the obvious way. We therefore have a recursion pattern that results in sequences of primes for multiple "starts".



Now for the cheat: there's no reason so far why this would make $b(i)$ positive. I don't have a completely elementary reason for that, but the prime number theorem should be enough to guarantee it. Essentially, the $n$th prime number doesn't grow "quickly enough" to escape such pairs.






share|cite|improve this answer









$endgroup$





















    0












    $begingroup$

    There are very good reasons to think that for every even $g$ there are infinitely many primes $p$ with $p+g$ also prime. I'll mention them at the end.



    For $g=10$ the primes with this property start out



    $$3, 7, 13, 19, 31, 37, 43, 61, 73, 79, 97, 103, 127, 139, 157, 163, 181, 223, 229,cdots$$



    If this is indeed infinite, then one can pick any sub-sequence and the shift by $10$ such as



    $$3,19,73,79,157,229,cdots$$
    $$13,29,83,89,167,239,cdots.$$



    Then $(A,B)$ is a recursion pattern for both sequences of primes where $A=1,1,1,cdots$ and $B=16,54,6,78,72,cdots$ is the correct $B$ sequence.



    There is known to be at least one $g leq 246$ with infinitely many primes $p,p+g.$ (in fact infinitely many with the two primes successive, not that we care here.) So that alone shows that the answer to your question is yes.



    Assuming what I said about even $g$ is true, if follows that there is great freedom in finding recursion patterns which describe two prime sequences. Specifically: suppose we play a game where you give me two odd primes $p_0 lt p_0'$ a multiplier $a_1$ and an integer $beta_1.$ Then I need to give you a shift $b_1 gt beta_1$ so that $p_1=a_1p_0+b_1$ and $p'_1=a_1p'_0+b_1$ are both prime. Then, after I tell you this, you can give me any $a_2$ and $beta_2$ and I must give $b_2 gt beta_2$ so that $p_2=a_2p_1+b_2$ and $p'_2=a_2p'_1+b_2$ are both prime. We continue in this way forever or until I get stuck.



    If I can always manage the first step of finding $b_1$ then I will never get stuck. It is just the same problem over and over with a different given $p,p',a,beta$ each time. Just pick $g=a_0(p'_0-p_0)$ and, since there are infinitely many pairs $p,p+g,$ just pick one with $p-a_1p_0 gt beta_1.$



    A theorem about prime constellations would suggest that one can similarly start with $s=3$ or $s=100$ initial primes and an arbitrary $A$ (revealed one step at a time) and pick $B$ to get $s$ prime sequences all with the same recursion pattern $(A,B)$



    Here are details on the claim at the top. Define $pi_g(x)$ to be the number of primes $p lt x$ with $p+g$ also prime. Then there are good simple reasons to expect that $pi_2(x)$ is asymptotic to $C_2frac{x}{(ln x)^2} $ for $C_2=Pi_{p geq 3}(1-frac1{(p-1)^2}).$ Numerical calculations up to high $x$ match this quite well. The same reasoning suggests that there is a similar $C_g$ for each $g$ which depends only on the prime divisors of $g$ So $C_{2^k}=C_2$ and for any $g=2^k3^j$ , $C_g=2C_2$. In general, $C_g=C_2Pi_{p mid g}frac{p-1}{p-2}.$ Again, the numerical evidence fits the prediction.






    share|cite|improve this answer









    $endgroup$













      Your Answer





      StackExchange.ifUsing("editor", function () {
      return StackExchange.using("mathjaxEditing", function () {
      StackExchange.MarkdownEditor.creationCallbacks.add(function (editor, postfix) {
      StackExchange.mathjaxEditing.prepareWmdForMathJax(editor, postfix, [["$", "$"], ["\\(","\\)"]]);
      });
      });
      }, "mathjax-editing");

      StackExchange.ready(function() {
      var channelOptions = {
      tags: "".split(" "),
      id: "504"
      };
      initTagRenderer("".split(" "), "".split(" "), channelOptions);

      StackExchange.using("externalEditor", function() {
      // Have to fire editor after snippets, if snippets enabled
      if (StackExchange.settings.snippets.snippetsEnabled) {
      StackExchange.using("snippets", function() {
      createEditor();
      });
      }
      else {
      createEditor();
      }
      });

      function createEditor() {
      StackExchange.prepareEditor({
      heartbeatType: 'answer',
      autoActivateHeartbeat: false,
      convertImagesToLinks: true,
      noModals: true,
      showLowRepImageUploadWarning: true,
      reputationToPostImages: 10,
      bindNavPrevention: true,
      postfix: "",
      imageUploader: {
      brandingHtml: "Powered by u003ca class="icon-imgur-white" href="https://imgur.com/"u003eu003c/au003e",
      contentPolicyHtml: "User contributions licensed under u003ca href="https://creativecommons.org/licenses/by-sa/3.0/"u003ecc by-sa 3.0 with attribution requiredu003c/au003e u003ca href="https://stackoverflow.com/legal/content-policy"u003e(content policy)u003c/au003e",
      allowUrls: true
      },
      noCode: true, onDemand: true,
      discardSelector: ".discard-answer"
      ,immediatelyShowMarkdownHelp:true
      });


      }
      });














      draft saved

      draft discarded


















      StackExchange.ready(
      function () {
      StackExchange.openid.initPostLogin('.new-post-login', 'https%3a%2f%2fmathoverflow.net%2fquestions%2f322322%2fcan-the-same-recursion-pattern-define-several-sequences-of-primes%23new-answer', 'question_page');
      }
      );

      Post as a guest















      Required, but never shown

























      2 Answers
      2






      active

      oldest

      votes








      2 Answers
      2






      active

      oldest

      votes









      active

      oldest

      votes






      active

      oldest

      votes









      4












      $begingroup$

      The answer is yes, there are "recursion patterns" which result in sequences of primes for more than one choice of $c(0)$. In fact, if you are given any two "starting points", it is possible to find such sequences. The proof comes from the pigeonhole principle.



      Start with $c(0), c'(0)$. These are two prime numbers with difference $d(1) := c'(0) - c(0)$. Then as the set of prime numbers is infinite, there is some pair $p_1 < p'_1$ such that $p_1 equiv p'_1 (text{mod }d)$. Let $a(1) = frac{p'_1 - p_1}{d(1)}$; let $b(1) = p_1 - a(1) c(0)$. (Note: I've slightly cheated here, and will explain the cheat at the end). Then $c(1) = a(1) c(0) + b(1) = a(1) c(0) + p_1 - a(1) c(0) = p_1$, while $c'(1) = a(1) c(0) + b(1) = frac{p'_1 - p_1}{c'(0) - c(0)} c'(0) + p_1 - frac{p'_1 - p_1}{c'(0) - c(0)} c(0) = p'_1$, as desired.



      This is a pattern that can be repeated in the obvious way. We therefore have a recursion pattern that results in sequences of primes for multiple "starts".



      Now for the cheat: there's no reason so far why this would make $b(i)$ positive. I don't have a completely elementary reason for that, but the prime number theorem should be enough to guarantee it. Essentially, the $n$th prime number doesn't grow "quickly enough" to escape such pairs.






      share|cite|improve this answer









      $endgroup$


















        4












        $begingroup$

        The answer is yes, there are "recursion patterns" which result in sequences of primes for more than one choice of $c(0)$. In fact, if you are given any two "starting points", it is possible to find such sequences. The proof comes from the pigeonhole principle.



        Start with $c(0), c'(0)$. These are two prime numbers with difference $d(1) := c'(0) - c(0)$. Then as the set of prime numbers is infinite, there is some pair $p_1 < p'_1$ such that $p_1 equiv p'_1 (text{mod }d)$. Let $a(1) = frac{p'_1 - p_1}{d(1)}$; let $b(1) = p_1 - a(1) c(0)$. (Note: I've slightly cheated here, and will explain the cheat at the end). Then $c(1) = a(1) c(0) + b(1) = a(1) c(0) + p_1 - a(1) c(0) = p_1$, while $c'(1) = a(1) c(0) + b(1) = frac{p'_1 - p_1}{c'(0) - c(0)} c'(0) + p_1 - frac{p'_1 - p_1}{c'(0) - c(0)} c(0) = p'_1$, as desired.



        This is a pattern that can be repeated in the obvious way. We therefore have a recursion pattern that results in sequences of primes for multiple "starts".



        Now for the cheat: there's no reason so far why this would make $b(i)$ positive. I don't have a completely elementary reason for that, but the prime number theorem should be enough to guarantee it. Essentially, the $n$th prime number doesn't grow "quickly enough" to escape such pairs.






        share|cite|improve this answer









        $endgroup$
















          4












          4








          4





          $begingroup$

          The answer is yes, there are "recursion patterns" which result in sequences of primes for more than one choice of $c(0)$. In fact, if you are given any two "starting points", it is possible to find such sequences. The proof comes from the pigeonhole principle.



          Start with $c(0), c'(0)$. These are two prime numbers with difference $d(1) := c'(0) - c(0)$. Then as the set of prime numbers is infinite, there is some pair $p_1 < p'_1$ such that $p_1 equiv p'_1 (text{mod }d)$. Let $a(1) = frac{p'_1 - p_1}{d(1)}$; let $b(1) = p_1 - a(1) c(0)$. (Note: I've slightly cheated here, and will explain the cheat at the end). Then $c(1) = a(1) c(0) + b(1) = a(1) c(0) + p_1 - a(1) c(0) = p_1$, while $c'(1) = a(1) c(0) + b(1) = frac{p'_1 - p_1}{c'(0) - c(0)} c'(0) + p_1 - frac{p'_1 - p_1}{c'(0) - c(0)} c(0) = p'_1$, as desired.



          This is a pattern that can be repeated in the obvious way. We therefore have a recursion pattern that results in sequences of primes for multiple "starts".



          Now for the cheat: there's no reason so far why this would make $b(i)$ positive. I don't have a completely elementary reason for that, but the prime number theorem should be enough to guarantee it. Essentially, the $n$th prime number doesn't grow "quickly enough" to escape such pairs.






          share|cite|improve this answer









          $endgroup$



          The answer is yes, there are "recursion patterns" which result in sequences of primes for more than one choice of $c(0)$. In fact, if you are given any two "starting points", it is possible to find such sequences. The proof comes from the pigeonhole principle.



          Start with $c(0), c'(0)$. These are two prime numbers with difference $d(1) := c'(0) - c(0)$. Then as the set of prime numbers is infinite, there is some pair $p_1 < p'_1$ such that $p_1 equiv p'_1 (text{mod }d)$. Let $a(1) = frac{p'_1 - p_1}{d(1)}$; let $b(1) = p_1 - a(1) c(0)$. (Note: I've slightly cheated here, and will explain the cheat at the end). Then $c(1) = a(1) c(0) + b(1) = a(1) c(0) + p_1 - a(1) c(0) = p_1$, while $c'(1) = a(1) c(0) + b(1) = frac{p'_1 - p_1}{c'(0) - c(0)} c'(0) + p_1 - frac{p'_1 - p_1}{c'(0) - c(0)} c(0) = p'_1$, as desired.



          This is a pattern that can be repeated in the obvious way. We therefore have a recursion pattern that results in sequences of primes for multiple "starts".



          Now for the cheat: there's no reason so far why this would make $b(i)$ positive. I don't have a completely elementary reason for that, but the prime number theorem should be enough to guarantee it. Essentially, the $n$th prime number doesn't grow "quickly enough" to escape such pairs.







          share|cite|improve this answer












          share|cite|improve this answer



          share|cite|improve this answer










          answered yesterday









          user44191user44191

          2,9681428




          2,9681428























              0












              $begingroup$

              There are very good reasons to think that for every even $g$ there are infinitely many primes $p$ with $p+g$ also prime. I'll mention them at the end.



              For $g=10$ the primes with this property start out



              $$3, 7, 13, 19, 31, 37, 43, 61, 73, 79, 97, 103, 127, 139, 157, 163, 181, 223, 229,cdots$$



              If this is indeed infinite, then one can pick any sub-sequence and the shift by $10$ such as



              $$3,19,73,79,157,229,cdots$$
              $$13,29,83,89,167,239,cdots.$$



              Then $(A,B)$ is a recursion pattern for both sequences of primes where $A=1,1,1,cdots$ and $B=16,54,6,78,72,cdots$ is the correct $B$ sequence.



              There is known to be at least one $g leq 246$ with infinitely many primes $p,p+g.$ (in fact infinitely many with the two primes successive, not that we care here.) So that alone shows that the answer to your question is yes.



              Assuming what I said about even $g$ is true, if follows that there is great freedom in finding recursion patterns which describe two prime sequences. Specifically: suppose we play a game where you give me two odd primes $p_0 lt p_0'$ a multiplier $a_1$ and an integer $beta_1.$ Then I need to give you a shift $b_1 gt beta_1$ so that $p_1=a_1p_0+b_1$ and $p'_1=a_1p'_0+b_1$ are both prime. Then, after I tell you this, you can give me any $a_2$ and $beta_2$ and I must give $b_2 gt beta_2$ so that $p_2=a_2p_1+b_2$ and $p'_2=a_2p'_1+b_2$ are both prime. We continue in this way forever or until I get stuck.



              If I can always manage the first step of finding $b_1$ then I will never get stuck. It is just the same problem over and over with a different given $p,p',a,beta$ each time. Just pick $g=a_0(p'_0-p_0)$ and, since there are infinitely many pairs $p,p+g,$ just pick one with $p-a_1p_0 gt beta_1.$



              A theorem about prime constellations would suggest that one can similarly start with $s=3$ or $s=100$ initial primes and an arbitrary $A$ (revealed one step at a time) and pick $B$ to get $s$ prime sequences all with the same recursion pattern $(A,B)$



              Here are details on the claim at the top. Define $pi_g(x)$ to be the number of primes $p lt x$ with $p+g$ also prime. Then there are good simple reasons to expect that $pi_2(x)$ is asymptotic to $C_2frac{x}{(ln x)^2} $ for $C_2=Pi_{p geq 3}(1-frac1{(p-1)^2}).$ Numerical calculations up to high $x$ match this quite well. The same reasoning suggests that there is a similar $C_g$ for each $g$ which depends only on the prime divisors of $g$ So $C_{2^k}=C_2$ and for any $g=2^k3^j$ , $C_g=2C_2$. In general, $C_g=C_2Pi_{p mid g}frac{p-1}{p-2}.$ Again, the numerical evidence fits the prediction.






              share|cite|improve this answer









              $endgroup$


















                0












                $begingroup$

                There are very good reasons to think that for every even $g$ there are infinitely many primes $p$ with $p+g$ also prime. I'll mention them at the end.



                For $g=10$ the primes with this property start out



                $$3, 7, 13, 19, 31, 37, 43, 61, 73, 79, 97, 103, 127, 139, 157, 163, 181, 223, 229,cdots$$



                If this is indeed infinite, then one can pick any sub-sequence and the shift by $10$ such as



                $$3,19,73,79,157,229,cdots$$
                $$13,29,83,89,167,239,cdots.$$



                Then $(A,B)$ is a recursion pattern for both sequences of primes where $A=1,1,1,cdots$ and $B=16,54,6,78,72,cdots$ is the correct $B$ sequence.



                There is known to be at least one $g leq 246$ with infinitely many primes $p,p+g.$ (in fact infinitely many with the two primes successive, not that we care here.) So that alone shows that the answer to your question is yes.



                Assuming what I said about even $g$ is true, if follows that there is great freedom in finding recursion patterns which describe two prime sequences. Specifically: suppose we play a game where you give me two odd primes $p_0 lt p_0'$ a multiplier $a_1$ and an integer $beta_1.$ Then I need to give you a shift $b_1 gt beta_1$ so that $p_1=a_1p_0+b_1$ and $p'_1=a_1p'_0+b_1$ are both prime. Then, after I tell you this, you can give me any $a_2$ and $beta_2$ and I must give $b_2 gt beta_2$ so that $p_2=a_2p_1+b_2$ and $p'_2=a_2p'_1+b_2$ are both prime. We continue in this way forever or until I get stuck.



                If I can always manage the first step of finding $b_1$ then I will never get stuck. It is just the same problem over and over with a different given $p,p',a,beta$ each time. Just pick $g=a_0(p'_0-p_0)$ and, since there are infinitely many pairs $p,p+g,$ just pick one with $p-a_1p_0 gt beta_1.$



                A theorem about prime constellations would suggest that one can similarly start with $s=3$ or $s=100$ initial primes and an arbitrary $A$ (revealed one step at a time) and pick $B$ to get $s$ prime sequences all with the same recursion pattern $(A,B)$



                Here are details on the claim at the top. Define $pi_g(x)$ to be the number of primes $p lt x$ with $p+g$ also prime. Then there are good simple reasons to expect that $pi_2(x)$ is asymptotic to $C_2frac{x}{(ln x)^2} $ for $C_2=Pi_{p geq 3}(1-frac1{(p-1)^2}).$ Numerical calculations up to high $x$ match this quite well. The same reasoning suggests that there is a similar $C_g$ for each $g$ which depends only on the prime divisors of $g$ So $C_{2^k}=C_2$ and for any $g=2^k3^j$ , $C_g=2C_2$. In general, $C_g=C_2Pi_{p mid g}frac{p-1}{p-2}.$ Again, the numerical evidence fits the prediction.






                share|cite|improve this answer









                $endgroup$
















                  0












                  0








                  0





                  $begingroup$

                  There are very good reasons to think that for every even $g$ there are infinitely many primes $p$ with $p+g$ also prime. I'll mention them at the end.



                  For $g=10$ the primes with this property start out



                  $$3, 7, 13, 19, 31, 37, 43, 61, 73, 79, 97, 103, 127, 139, 157, 163, 181, 223, 229,cdots$$



                  If this is indeed infinite, then one can pick any sub-sequence and the shift by $10$ such as



                  $$3,19,73,79,157,229,cdots$$
                  $$13,29,83,89,167,239,cdots.$$



                  Then $(A,B)$ is a recursion pattern for both sequences of primes where $A=1,1,1,cdots$ and $B=16,54,6,78,72,cdots$ is the correct $B$ sequence.



                  There is known to be at least one $g leq 246$ with infinitely many primes $p,p+g.$ (in fact infinitely many with the two primes successive, not that we care here.) So that alone shows that the answer to your question is yes.



                  Assuming what I said about even $g$ is true, if follows that there is great freedom in finding recursion patterns which describe two prime sequences. Specifically: suppose we play a game where you give me two odd primes $p_0 lt p_0'$ a multiplier $a_1$ and an integer $beta_1.$ Then I need to give you a shift $b_1 gt beta_1$ so that $p_1=a_1p_0+b_1$ and $p'_1=a_1p'_0+b_1$ are both prime. Then, after I tell you this, you can give me any $a_2$ and $beta_2$ and I must give $b_2 gt beta_2$ so that $p_2=a_2p_1+b_2$ and $p'_2=a_2p'_1+b_2$ are both prime. We continue in this way forever or until I get stuck.



                  If I can always manage the first step of finding $b_1$ then I will never get stuck. It is just the same problem over and over with a different given $p,p',a,beta$ each time. Just pick $g=a_0(p'_0-p_0)$ and, since there are infinitely many pairs $p,p+g,$ just pick one with $p-a_1p_0 gt beta_1.$



                  A theorem about prime constellations would suggest that one can similarly start with $s=3$ or $s=100$ initial primes and an arbitrary $A$ (revealed one step at a time) and pick $B$ to get $s$ prime sequences all with the same recursion pattern $(A,B)$



                  Here are details on the claim at the top. Define $pi_g(x)$ to be the number of primes $p lt x$ with $p+g$ also prime. Then there are good simple reasons to expect that $pi_2(x)$ is asymptotic to $C_2frac{x}{(ln x)^2} $ for $C_2=Pi_{p geq 3}(1-frac1{(p-1)^2}).$ Numerical calculations up to high $x$ match this quite well. The same reasoning suggests that there is a similar $C_g$ for each $g$ which depends only on the prime divisors of $g$ So $C_{2^k}=C_2$ and for any $g=2^k3^j$ , $C_g=2C_2$. In general, $C_g=C_2Pi_{p mid g}frac{p-1}{p-2}.$ Again, the numerical evidence fits the prediction.






                  share|cite|improve this answer









                  $endgroup$



                  There are very good reasons to think that for every even $g$ there are infinitely many primes $p$ with $p+g$ also prime. I'll mention them at the end.



                  For $g=10$ the primes with this property start out



                  $$3, 7, 13, 19, 31, 37, 43, 61, 73, 79, 97, 103, 127, 139, 157, 163, 181, 223, 229,cdots$$



                  If this is indeed infinite, then one can pick any sub-sequence and the shift by $10$ such as



                  $$3,19,73,79,157,229,cdots$$
                  $$13,29,83,89,167,239,cdots.$$



                  Then $(A,B)$ is a recursion pattern for both sequences of primes where $A=1,1,1,cdots$ and $B=16,54,6,78,72,cdots$ is the correct $B$ sequence.



                  There is known to be at least one $g leq 246$ with infinitely many primes $p,p+g.$ (in fact infinitely many with the two primes successive, not that we care here.) So that alone shows that the answer to your question is yes.



                  Assuming what I said about even $g$ is true, if follows that there is great freedom in finding recursion patterns which describe two prime sequences. Specifically: suppose we play a game where you give me two odd primes $p_0 lt p_0'$ a multiplier $a_1$ and an integer $beta_1.$ Then I need to give you a shift $b_1 gt beta_1$ so that $p_1=a_1p_0+b_1$ and $p'_1=a_1p'_0+b_1$ are both prime. Then, after I tell you this, you can give me any $a_2$ and $beta_2$ and I must give $b_2 gt beta_2$ so that $p_2=a_2p_1+b_2$ and $p'_2=a_2p'_1+b_2$ are both prime. We continue in this way forever or until I get stuck.



                  If I can always manage the first step of finding $b_1$ then I will never get stuck. It is just the same problem over and over with a different given $p,p',a,beta$ each time. Just pick $g=a_0(p'_0-p_0)$ and, since there are infinitely many pairs $p,p+g,$ just pick one with $p-a_1p_0 gt beta_1.$



                  A theorem about prime constellations would suggest that one can similarly start with $s=3$ or $s=100$ initial primes and an arbitrary $A$ (revealed one step at a time) and pick $B$ to get $s$ prime sequences all with the same recursion pattern $(A,B)$



                  Here are details on the claim at the top. Define $pi_g(x)$ to be the number of primes $p lt x$ with $p+g$ also prime. Then there are good simple reasons to expect that $pi_2(x)$ is asymptotic to $C_2frac{x}{(ln x)^2} $ for $C_2=Pi_{p geq 3}(1-frac1{(p-1)^2}).$ Numerical calculations up to high $x$ match this quite well. The same reasoning suggests that there is a similar $C_g$ for each $g$ which depends only on the prime divisors of $g$ So $C_{2^k}=C_2$ and for any $g=2^k3^j$ , $C_g=2C_2$. In general, $C_g=C_2Pi_{p mid g}frac{p-1}{p-2}.$ Again, the numerical evidence fits the prediction.







                  share|cite|improve this answer












                  share|cite|improve this answer



                  share|cite|improve this answer










                  answered 1 hour ago









                  Aaron MeyerowitzAaron Meyerowitz

                  23.9k13287




                  23.9k13287






























                      draft saved

                      draft discarded




















































                      Thanks for contributing an answer to MathOverflow!


                      • Please be sure to answer the question. Provide details and share your research!

                      But avoid



                      • Asking for help, clarification, or responding to other answers.

                      • Making statements based on opinion; back them up with references or personal experience.


                      Use MathJax to format equations. MathJax reference.


                      To learn more, see our tips on writing great answers.




                      draft saved


                      draft discarded














                      StackExchange.ready(
                      function () {
                      StackExchange.openid.initPostLogin('.new-post-login', 'https%3a%2f%2fmathoverflow.net%2fquestions%2f322322%2fcan-the-same-recursion-pattern-define-several-sequences-of-primes%23new-answer', 'question_page');
                      }
                      );

                      Post as a guest















                      Required, but never shown





















































                      Required, but never shown














                      Required, but never shown












                      Required, but never shown







                      Required, but never shown

































                      Required, but never shown














                      Required, but never shown












                      Required, but never shown







                      Required, but never shown







                      Popular posts from this blog

                      If I really need a card on my start hand, how many mulligans make sense? [duplicate]

                      Alcedinidae

                      Can an atomic nucleus contain both particles and antiparticles? [duplicate]